Schrodinger's Cat.

New Topic
This topic has been archived, and won't accept reply postings.

I'm hoping to tune in to some of UKC's finest...

Could you help me out and explain..The Dead Cat/Alive Cat theory..

Thankyou

 marsbar 11 Feb 2021
In reply to Shaun mcmurrough:

Its a thought experiment.  The idea being that the cat is killed or not killed depending on the result of an experiment with an exact probability of 1/2 or 50 50 chance.  

Without opening the box you don't know if the cat is alive or dead.  

So the cat is considered in an indeterminate state until you open the box.

There is more to it which is the important part to do with not being able to tell both the position and velocity of a particle.  

I will have to look it up to make sure I don't explain it wrong. 

Post edited at 14:31
1
 marsbar 11 Feb 2021
In reply to marsbar:

This is how Schrodinger explains it. 

> One can even set up quite ridiculous cases. A cat is penned up in a steel chamber, along with the following device (which must be secured against direct interference by the cat): in a Geiger counter, there is a tiny bit of radioactive substance, so small, that perhaps in the course of the hour one of the atoms decays, but also, with equal probability, perhaps none; if it happens, the counter tube discharges and through a relay releases a hammer that shatters a small flask of hydrocyanic acid. If one has left this entire system to itself for an hour, one would say that the cat still lives if meanwhile no atom has decayed. The first atomic decay would have poisoned it. The psi-function of the entire system would express this by having in it the living and dead cat (pardon the expression) mixed or smeared out in equal parts.

Post edited at 14:41
 marsbar 11 Feb 2021
In reply to marsbar:

So it's a thought process that has practical applications then?

 marsbar 11 Feb 2021
In reply to Shaun mcmurrough:

He was I think trying to show how the way scientists explained things didn't really make sense.  

The idea that something doesn't happen until its observer makes it happen.  

Having a cat that isn't dead or alive but somehow is both doesn't make sense. 

I guess it's a more complicated version of does a tree falling make a sound if no one hears it.

Hopefully someone with a much better grasp on Physics will come and explain it better.  It's an interesting topic but I don't think I really understand it beyond the basics.  

In reply to marsbar:

I appreciate the input....I read a bit and watched a bit and it's really much the same language...the tree falling,has been used a bit on here.

I need to be in a classroom so someone can check my understanding as it's discussed..

cb294 11 Feb 2021
In reply to Shaun mcmurrough:

It illustrates that there is a point when quantum mechanics, which works like a dream at the micro scale* starts giving nonsensical or at least counterintuitive results when moving to more macro scales.

Why this should be so is unclear, and still, decades after the thought experiment, a core problem of physics.**

As for practical applications, almost no current technology involving anything at the micro or nano scale can be understood or improved without quantum mechanics. Any progresss in our understanding of the theoretical basis of that theory will likely lead to new technology, even if no one can predict exactly what will come out of such novel insights, which by definition we do not yet understand.

Zombie cats as such are not useful!

CB

* as evidenced by the electronics of the computer I am typing this on, by our understanding of chemistry, etc.

** edit to include: this is probably a sign that quantum mechanics is not the final theory here. Another hint for this is that it is incompatible with Einstein's theory of general relativity, which suffers from the opposite problem, by giving perfect results at large scales, but failing as you get to small scales where QM works nicely!

Post edited at 15:00
 john arran 11 Feb 2021
In reply to Shaun mcmurrough:

The tree falling isn't quite the same as there are physical laws that 'prove' that a sound will still be made. In particle superposition, the state of the particle is fundamentally unknowable, since no law of physics can deterministically predict it.

1
 tlouth7 11 Feb 2021
In reply to Shaun mcmurrough:

The thing to keep in mind is that this thought experiment was specifically conceived to demonstrate how the things that we accept at a quantum (very small) scale are absurd at a macro (everyday) scale.

A foundation of quantum mechanics is the idea that systems exist in every possible state until they are observed, at which point they sort of randomly 'pick' which of the possible states they are in. There are various very elegant real world experiments that demonstrate this effect. For example the double slit experiment but with one photon at a time.

What Schrodinger did was propose a mechanism that would in effect scale up this quantum effect to the macro scale. Say you have a lump of radioactive substance with a 50% chance of having a single decay event (and so emitting a single electron) and you don't observe it. The prevailing quantum theory of the time would tell us that the system exists in both states, decayed and not decayed. This is hard for a layperson to wrap their head around but accepted in the world of quantum mechanics.

Now imagine a cat in the box, and some mechanism to kill the cat if a decay event occurs. We still can't observe the possible decay event, so nothing has changed about that. The superposition of both possible states is still there. But if both decay and no decay really physically exist, then it follows that the cat is both alive and dead in a real physical way (perhaps I have moved through that step a bit fast?). Schrodinger was arguing that this is patently absurd, but is it?

 Graeme G 11 Feb 2021
In reply to Shaun mcmurrough:

Repression (movie on Sky) uses this as its base idea. It’s ok, I enjoyed it. No spoilers but in one sequence the boy states there is a gun in a drawer. The only way you know the gun is in the drawer is to open it. Question - was the gun already there, or did your opening the drawer cause the gun to appear?

1
 Lankyman 11 Feb 2021
In reply to Shaun mcmurrough:

> I'm hoping to tune in to some of UKC's finest...

> Could you help me out and explain..The Dead Cat/Alive Cat theory..

> Thankyou


By chance, I'm almost at the end of 'In Search of Schrödinger's Cat' by John Gribbin. I thought I'd better try and plumb some of the depths of my ignorance before I'm completely gaga. I can safely say that I'm still officially 'thick as pigsh1t' as far as quantum mechanics goes. Reality is very odd indeed and it seems to hinge on whether the Copenhagen interpretation or the many-worlds interpretation is the true picture. What's not known is WHY the cat went into the box in the first place. Curiosity may (or may not) kill the cat.

 Sean Kelly 11 Feb 2021
In reply to Lankyman:

What's  not known is why the cat went into the box in the first place?

Because it's there!

 DerwentDiluted 11 Feb 2021
In reply to Shaun mcmurrough:

There is a variation on this experiment. Bales Cat.

If you put a cat in a wheelie bin, and don't see the cctv camera,  there is an unknowable 50-50 chance that the whole thing will be recorded, watched, go viral on the internet and your reputation will be killed. And so in the hour following putting the cat in the bin, her lifelong future reputation was both alive, and dead.

Post edited at 15:48
 Lankyman 11 Feb 2021
In reply to Sean Kelly:

> What's  not known is why the cat went into the box in the first place?

> Because it's there!

If the cyanide in the box was wrapped in an old butter packet my old cat Charlie would have thought 50/50 survival chance pretty good odds.

 GerM 11 Feb 2021
In reply to Shaun mcmurrough:

I'll assume it's both, until you can show me otherwise.

 wercat 11 Feb 2021
In reply to Shaun mcmurrough:

Think of the Cat, or instead think of a bit of Tat, Schroedinger's Tat.

You'll find it in superposed states of being sound and unsound simultaneously above a huge drop - you'll feel the superposition as you sweat about it's actual state.

That is no good though, it's only after you've clipped on the rope and swing out to load it that you will force it into an actual state of being safe or not safe.  The strange thing is that the perception you receive of the actual state, safe or unsafe will always last for the same proportion of your remaining life - 100%.  However the actual time you experience this knowledge differs vastly between the safe and unsafe result

Post edited at 19:15
 wercat 11 Feb 2021
In reply to Lankyman:

I started the Gribbin Book in the 1980s.  I like it but I have to say that it stopped making sense and I gave up on it after a while.

In the early 2000s I got hold of a copy of "A Brief History of Time" by You Know Who.  There was so much I could not follow and after a while I decided that there might be something wrong.   Too much left out so it didn't progress a step at a time and things did not tie up.

I started a long period of reading - starting with The Mysterious Universe, written around 1930 by Sir James Jeans which summarised contemporary states of knowledge on physics and cosmology about that time without any inaccurate modern mis-summarisation.  I progressd through various books including Marcus Chown on "The Afterglow of Creation", lots of Brian Greene (The Fabric of the Cosmos, The Elegant Universe), Feynman's QED, books on "The Nature of Matter" and "The nature of Space" by Joan Solomon in the 1970s and lots of others written through the decades.   It was possible to piece together knowledge states for various time periods leading up to Hawking and Gribbin and fill in the gaps and in particular with Hawking the non sequiturs and incomplete statements and summaries about previous knowledge.  The result was I was able to read a Brief History of Time eventually and be a bit happier with it, and,

I finally finished Gribbin around 2006!  I know it was then because I had a tour round the enormous Einstein exhibition in Berne to commemmorate 1905 in 2005 and had a tour on my own with a guide as I was the only English person needing a tour that afternoon.  So the guided tour led to a room where the resident physicist was lurking and I was able to question him and discuss all this stuff in the context of the exhibition and this led to reading the whole Gribbin!

try to get hold of some of those books and have patience - each book gives you a different view of bits of the jigsaw.  Solomon, Chown and Greene are really excellent and you can read Greene several times and pick up a bit more each time.  Really worth it for what you can ponder before you close your eyes  at night.

The Jeans book is interesting as it is from the time when a lot of the quantum world was quite new.

Post edited at 19:39
 Lankyman 11 Feb 2021
In reply to wercat:

It's not taking me quite as long to get through Gribbin (months not decades!) but it's still been a bit of a slog for me. I tried reading Hawkings years ago but gave up on him and have no desire to return. It gives me some comfort that the universe is so strange that even the experts can't fully grasp it all.

 Cobra_Head 11 Feb 2021
In reply to Sean Kelly:

> What's  not known is why the cat went into the box in the first place?

> Because it's there!


Cat's love boxes though, don't they?

I've had one in a box for the last two years, never heard a peep out of him, he must be very happy I'd say.

 Michael Hood 11 Feb 2021
In reply to Shaun mcmurrough:

IIRC there's a saying something like "anyone who tells you they totally understand quantum mechanics is lying" - probably.

 Shani 11 Feb 2021
In reply to Shaun mcmurrough:

I don't think the explanations given thus far give a good context - so this is how i understand it's significance;

First of all, are you aware of the double slit experiment?  youtube.com/watch?v=A9tKncAdlHQ&

The crucial thing to note is that even when releasing a single photon at a time you still get an interference pattern; thus one photon is passing through BOTH slits (and interfering with itself). This is the superposition bit others have mentioned above.

But, if you put a detector on either or both slits and repeat the experiment with a single photon at a time, the photon only goes through one OR the other slit, you don't get the interference pattern.

It's as if the photon knows its being watched so behaves 'normally'. The act of merely watching the photon stops the superposition happening (collapse the wave function).

This is where Shroedinger's Cat comes in to it. If you could rig up two slits in a box with a cat and had a trigger on one slit to kill the cat, and, a trigger on the other slit to release the cat, what would happen? The photon goes through BOTH slits so the cat is both killed and released at the same time (notwithstanding the fact above that any sensor would collapse the wave function - but you get the idea).

And so it is only by opening the box and looking at the photon that the photon has to go through one OR the other slit so we definitely know the fate of the cat.

I hope that illustrates the weirdness.

In reply to Shani:

I was reading all the replies with yours being last....I feel I might be just be starting to understand...

My wife has just questioned what I'm doing.....her response..'didn't you know that...a photon is a fake'...🤪

Post edited at 21:40
 Shani 11 Feb 2021
In reply to Shaun mcmurrough:

I've taken a few liberties with my example so it's not exactly correct (as i explained, you couldn't have sensors on the slits to trigger the kill/release as this collapses the wave function/superposition), but despite that, it does tie up the quantum weirdness between cats and the double slit experiment. 

The basic double slit experiment is easy to set up and demonstrate - even at home. But when you find out that you get the same result using single photons at a time - whoa. Mind. Blown.

The thing is, without life in the universe to observe stuff and so collapse the wave function does EVERYTHING exist in a superposition? Was there a point when the first 'observer' evolved and 'boom' everything collapsed down?

This thought troubles me daily.

Post edited at 21:52
 wercat 11 Feb 2021
In reply to Shani:

I believe that when particles are in contact with other particles to form matter on our scale then measurement is intrinsic to the relationships of the particles with each other - measurement is the observation phrased more usefully.  In guaging each others quantum states results in the matter behaving properly as we expect on the macro scale - the need for a conscious observer is a misunderstanding

 Jon Stewart 11 Feb 2021
In reply to Shani:

> The thing is, without life in the universe to observe stuff and so collapse the wave function does EVERYTHING exist in a superposition? Was there a point when the first 'observer' evolved and 'boom' everything collapsed down?

The collapse of the wavefunction doesn't rely on a conscious observer. 

In the double-slit experiment, any detector that told you which slit the particle (if you're doing it with say electrons) or photon went through would have to interact with that particle (or photon). The wavefunction is completely different in the detector case than it is in the no detector case. No conscious being needs to be looking at the detector to collapse the wavefunction. It's still weird because there aren't good criteria for what collapses the wavefunction (if you don't believe in the many worlds version), but the idea that it's got something to do with conscious observers is bunk.

Schrondinger's Cat illustrates well that we don't know what the criteria are - I don't think anyone really believes that the cat goes from being in a superposition to being in one state when the observer looks inside.

Edit. As wercat said, I think.

Post edited at 22:25
 Shani 11 Feb 2021
In reply to wercat:

> I believe that when particles are in contact with other particles to form matter on our scale then measurement is intrinsic to the relationships of the particles with each other - measurement is the observation phrased more usefully.  In guaging each others quantum states results in the matter behaving properly as we expect on the macro scale - the need for a conscious observer is a misunderstanding

Thanks for that. It makes sense and assuages my mind!

 Shani 11 Feb 2021
In reply to Jon Stewart:

Again, yeah that makes sense. So for my understanding, what is an example of the most 'basic' detector? What non-human derived thing(s) would collapse the wave function?

 Jon Stewart 11 Feb 2021
In reply to Shani:

> Again, yeah that makes sense. So for my understanding, what is an example of the most 'basic' detector? What non-human derived thing(s) would collapse the wave function?

If you wanted to know which slit an electron went through, I guess you'd probably shine a load of photons or something in that general direction and look for the interaction. So just a whole load more particles in the system, one interacting with the electron of interest, collapses the wavefunction in this case.

But in the case of the cat, we don't know, which is the point.

 Weekend Punter 11 Feb 2021
In reply to Shaun mcmurrough:

> So it's a thought process that has practical applications then?

One of the main points of the experiment is that the observer affects the outcome of the experiment at the time of observing the result. Also at the moment the cat is observed to be either dead or alive it instantly isn't alive or isn't dead respectively. Better to think that two people have bet differently on the state of the cat, only at the time the cat is observed does one win and instantaneously one loses.

It's quite a restrictive experiment in that the cat won't live indefinitely and the two states (dead/not alive or alive/not dead) are observed in the same location (the box). As I understand it, the concept of quantum entanglement is an expansion on this experiment and lifts the restriction of observing either state and it's counter state in the same location. This in theory allows for data to be transferred faster than the speed of light with one application being a quantum computing network.

After dusting off two decade old physics tuition there could be some gaps in the explanation... but only if someone observes it to be!

 Misha 11 Feb 2021
In reply to Shaun mcmurrough:

The winter route in a long lost, far away Corrie in the North West is either in condition or it's not. To all intents and purposes, it's both at the same time, until you rock up to have a look, at which point it will be either in or not in. However in practice you'll never find out due to Covid restrictions so Schroedinger's Route continues in its undetermined state for the time being.

In reply to Jon Stewart:

> The collapse of the wavefunction doesn't rely on a conscious observer. 

> In the double-slit experiment, any detector that told you which slit the particle (if you're doing it with say electrons) or photon went through would have to interact with that particle (or photon). The wavefunction is completely different in the detector case than it is in the no detector case. No conscious being needs to be looking at the detector to collapse the wavefunction. It's still weird because there aren't good criteria for what collapses the wavefunction (if you don't believe in the many worlds version), but the idea that it's got something to do with conscious observers is bunk.

> Schrondinger's Cat illustrates well that we don't know what the criteria are - I don't think anyone really believes that the cat goes from being in a superposition to being in one state when the observer looks inside.

> Edit. As wercat said, I think.

You say this as if your interpretation is the settled interpretation of quantum mechanics but this debate has been ongoing for a century and is still one of the most debated questions in science.

As I see it, whether the wavefunction collapses due to an observation that is made by an effect directly registered by a human eye or whether the observation is recorded by some sort of detector, the common factor in both is that the information about what has happened is now available to a conscious observer.

 alexm198 12 Feb 2021
In reply to Jon Stewart:

> the idea that it's got something to do with conscious observers is bunk.

As you rightly point out, we don't actually know what the collapse mechanism is (if indeed there is one). So, unpalatable though they might be, surely this means mechanisms that rely on conscious observation are still on the table. I definitely agree they're deeply unfashionable and philosophically dubious but it seems a bit much to accuse people like Penrose, von Neumann and Wigner of peddling 'bunk' interpretations of QM.  

Earnest question: do you think conscious collapse theories are any more fanciful than Everettian ones?

> If you wanted to know which slit an electron went through, I guess you'd probably shine a load of photons or something in that general direction and look for the interaction. So just a whole load more particles in the system, one interacting with the electron of interest, collapses the wavefunction in this case.

I agree in principle but you stop short of the real philosophical problem.

The process you describe does not, when modelled using the apparatus of ordinary QM, collapse the wavefunction. All the Schrödinger equation has to say is that you'll simply end up with a larger superposition of coupled system/detector measurement states.

I can't work out if you're simply appealing to the intuition that as particle number becomes very large we expect classical dynamics to emerge from an underlying quantum description (in which case yes, but showing how is precisely what's at stake) or whether you're presuming some sort of decoherence or dynamical collapse mechanism (not itself uncontroversial). Either way, you talk as if this settles the matter but I don't see how that could be the case. 

 Michael Hood 12 Feb 2021
In reply to cumbria mammoth:

My understanding is that we don't actually know what atoms, photons, electrons, etc are.

They can exhibit particle behaviour, and they can exhibit wave behaviour. Both of those are things we understand from our macro level experience, but we're just assigning those to these entities because that's the best we can do. And that leads to these quantum paradoxes like the double slit experiment.

If we truly understood these entities then these paradoxes would disappear, however such understanding may be beyond human limits.

 tlouth7 12 Feb 2021
In reply to Weekend Punter:

> As I understand it, the concept of quantum entanglement is an expansion on this experiment and lifts the restriction of observing either state and it's counter state in the same location. This in theory allows for data to be transferred faster than the speed of light with one application being a quantum computing network.

I'm afraid entanglement doesn't allow faster than light communication. I have a good analogy for that if it would be of interest.

 1poundSOCKS 12 Feb 2021
In reply to Jon Stewart:

> Schrondinger's Cat illustrates well that we don't know what the criteria are - I don't think anyone really believes that the cat goes from being in a superposition to being in one state when the observer looks inside.

I heard that the problem with trying to get your head around SC was that they were treating the observer as classical, not part of the quantum system. Something to do with entanglement, which is maybe what the confusion over what the observer is comes from? So you get the many worlds interpretation, where you get one universe where the observer sees a live cat, and one universe where the observer see the dead cat. There's a split. Leads to a lot of universes. Some other physicists think it's a cop out.

 john arran 12 Feb 2021
In reply to 1poundSOCKS:

> Some other physicists think it's a cop out.

Given that it's fundamentally unprovable, it's not unlike invoking God.

 Phil Lyon 12 Feb 2021
In reply to wercat:

> In the early 2000s I got hold of a copy of "A Brief History of Time" by You Know Who. 

Voldemort?

I was thinking about Schrodinger's Alarm Clock this morning. You have no idea what the time is, the night could be endless, but as soon as you look at the alarm clock, it's about to go off.

 1poundSOCKS 12 Feb 2021
In reply to john arran:

> Given that it's fundamentally unprovable, it's not unlike invoking God.

Sounds a bit of a cop out to me. Sean Carroll seems to like it, so what do I know? Sabine Hossenfelder aint having none of it though.

EDIT: But even without the multi worlds bit, the idea of observer and observed being explained because they're entangled, not because you need a conscious observer, does kind of make sense to a non physicist.

Post edited at 10:23
 Jon Stewart 12 Feb 2021
In reply to john arran:

> Given that it's fundamentally unprovable, it's not unlike invoking God.

That's the exact objection to "consciousness collapse". How can you have a physical theory that invokes something for which there is no physical theory?

"Consciousness did it" makes as much sense in physics as "God did it". As it happens, I'm a realist with respect to consciousness, but not God, but physics as nothing to say about either. 

 alimckay 12 Feb 2021
In reply to Shaun mcmurrough:

There’s a good episode of the BBC radio 4 programme The Infinite Monkey Cage about this subject. Thoroughly recommend it if you have any interest in science. Mixes science with comedy.

https://www.bbc.co.uk/sounds/play/m000f5hl

 alexm198 12 Feb 2021
In reply to john arran:

But there's a strong sense in which the many worlds interpretation is actually the least fanciful interpretation of them all. It deals with the fact that the dynamics of quantum theory seem to describe 'all of the structure of |dead cat>' and 'all of the structure of |alive cat>' by simply saying, "hey, the theory tells us that both things are happening, I guess that's kind of odd but we should probably trust the theory".

A proponent of many worlds could turn around and say that invoking a completely unjustified, non-unitary collapse mechanism as a deus ex machina to avoid having to stomach the idea that physics on the small scale appears to contradict our empirical experience (that is, invoking collapse to save the primacy of human experience) seems much more god-invoking. 

Post edited at 12:07
In reply to Michael Hood:

> My understanding is that we don't actually know what atoms, photons, electrons, etc are.

> They can exhibit particle behaviour, and they can exhibit wave behaviour. Both of those are things we understand from our macro level experience, but we're just assigning those to these entities because that's the best we can do. And that leads to these quantum paradoxes like the double slit experiment.

> If we truly understood these entities then these paradoxes would disappear, however such understanding may be beyond human limits.

I agree and would go further.

My interpretation of all this is that our human experience of 'reality' is just a model constructed by our brains that helps us to compete with other animals but does not necessarily correspond well with how things really are. The brain filters out any sensory information that doesn't fit the model as this would hinder our ability to compete for the resources we need to survive.

We assign wave behaviour or particle behaviour, and I'd add time as a concept that physics tells us may not exist as we experience it, to things because that is how our simplified brain model of reality is constructed. I think that particles, waves, and time, may not necessarily be real features of the physical world at all but just a useful model created by our brain.

 Weekend Punter 12 Feb 2021
In reply to tlouth7:

> I'm afraid entanglement doesn't allow faster than light communication. I have a good analogy for that if it would be of interest.

Yes please share if you don't mind and I'll take a read

In reply to alexm198:

> But there's a strong sense in which the many worlds interpretation is actually the least fanciful interpretation of them all. It deals with the fact that the dynamics of quantum theory seem to describe 'all of the structure of |dead cat>' and 'all of the structure of |alive cat>' by simply saying, "hey, the theory tells us that both things are happening, I guess that's kind of odd but we should probably trust the theory".

> A proponent of many worlds could turn around and say that invoking a completely unjustified, non-unitary collapse mechanism as a deus ex machina to avoid having to stomach the idea that physics on the small scale appears to contradict our empirical experience (that is, invoking collapse to save the primacy of human experience) seems much more god-invoking. 

It does seem like the proponents of the many worlds are invoking an unjustified collapse mechanism to avoid having to stomach the idea of invoking God. 

This doesn't strike me as being the rational interpretation, if occams razor is pointing towards conscious observers having a fundamental role in the nature of reality then that really ought to be the dominant interpretation. 

I don't see it as leading to needing God to explain it anyway, consciousness as a fundamental property of the universe needs God as an explanation no more or less than the existence of the universe itself. Neither the multiverse nor the single universe with consciousness as a fundamental property will prove or disprove God.

 alexm198 12 Feb 2021
In reply to cumbria mammoth:

But the wavefunction doesn't collapse in many worlds. My point was that a proponent of many worlds would claim that stipulating some dubious collapse mechanism to save our metaphysical intuitions is more radical than simply taking the quantum formalism at face value (which is what many worlds proponents take themselves to be doing).

 wercat 12 Feb 2021
In reply to cumbria mammoth:

I believe in radio waves!  They reflect off the ionosphere and they can't just be particles as you would have trouble modulating one over time with, say, a Nigel Farage broadcast.  Accelerate an electron back and forth (point of presence of the electric and magnetic fields, probably a wave too but only chasing its own tail at the speed of light rather than travelling)  and the fields waggle too

plus the idea of our brains filtering out things that don't fit into our reality is totally unnecessary - our perceptions are based on what our real bodies and component materials can perceive which is levels of abstraction above the quantum level - so it's the only way they can perceive as it is the way they work - no filtering is necessary

Post edited at 16:13
 mike123 12 Feb 2021
In reply to Shaun mcmurrough: mainly agree what has been said so far . The important starting point is that it  a thought experiment , the cat can’t be alive and dead . It’s one or the other . Schrodinger was unhappy with the prevailing thinking on quantum mechanics which was the Copenhagen interpretation ( read about this )  , but Neil’s Bohr and Werner  heisenburg ( not the meth cook , he was a very naughty boy ) were well established and much more respected than schrodinger so he struggled  to convince people . The hard thing to grasp is that things , cans of beer , big bags of Doritos , lemon drizzle cakes , iPhones etc are made of atoms and molecules . They are in turn are made of protons neutrons and electrons . So electrons should behave the same as cans of beer etc ?  But they don’t . Electrons don’t behave according to the laws of natural logic . Far from it . See reference above to youngs double slit experiment and wave particle duality . There is some good stuff on you tube . I like this one but there are lots of others : 

youtube.com/watch?v=OkVpMAbNOAo&

whats slightly sad I

is that Ernest schrodinger is famous for this one little thing where as his contribution to quantum mechanics / physics was huge . Schrodinger s wave equation is beautifully elegant and modern physics wouldn’t have developed as it has without it .

 Jon Stewart 12 Feb 2021
In reply to alexm198:

> As you rightly point out, we don't actually know what the collapse mechanism is (if indeed there is one). So, unpalatable though they might be, surely this means mechanisms that rely on conscious observation are still on the table. I definitely agree they're deeply unfashionable and philosophically dubious but it seems a bit much to accuse people like Penrose, von Neumann and Wigner of peddling 'bunk' interpretations of QM.  

Penrose links collapse of the wavefunction to consciousness, but doesn't claim that consciousness causes collapse. Don't know exactly what the others say. Trouble with those guys is that they're not getting anywhere. If their work was making any progress I would agree with you. I really like David Chalmers and he's interested in linking the hard problem of consciousness with the hard problem of interpreting QM - but he's not getting anywhere.

Loads of clever people spend their whole lives pursuing theories that are obviously bunk to the rest of us, because they really want them to be true, e.g. all theologians, economists, etc.

> Earnest question: do you think conscious collapse theories are any more fanciful than Everettian ones?

Yes. Conscious collapse is proposing a physical process, which appears utterly improbable (*how* could consciousness collapse the wavefunction? Whose consciousness?). Many worlds is just saying: there's no additional process, reality is just arranged in a peculiar way described by the wavefunction of the universe. It's very odd, but it's not proposing anything additional, it's not invoking something about which we have no physics to make physics work.

> I agree in principle but you stop short of the real philosophical problem.

> The process you describe does not, when modelled using the apparatus of ordinary QM, collapse the wavefunction. All the Schrödinger equation has to say is that you'll simply end up with a larger superposition of coupled system/detector measurement states.

That's fair. So we've got a few options: we could go for many worlds; or we could say that something about the system where you're making measurements collapses the wavefunction, or you could say that the whole system stays in superposition until a conscious observer notices. The first 2 options are reasonable and are either described by our current physics (but weird) or could be described by some new unknown physics. The third option is the same as the second, except that it's constrained by tying itself to consciousness, something about which we have no physics. Seems like an obviously bunk idea to me.

> I can't work out if you're simply appealing to the intuition that as particle number becomes very large we expect classical dynamics to emerge from an underlying quantum description (in which case yes, but showing how is precisely what's at stake) or whether you're presuming some sort of decoherence or dynamical collapse mechanism (not itself uncontroversial). Either way, you talk as if this settles the matter but I don't see how that could be the case. 

I'm just saying that we don't know what the collapse mechanism is (if there is one), but the idea of consciousness having a role is totally silly. You don't bring in entities or properties from outside physics (consciousness, god, magic, etc) into theories of physics to make them more accurate. If we can bring consciousness into physics by solving the hard problem of consciousness (as Penrose has attempted), then fine, that would put the option sensibly on the table. As things stands, you may as well appeal to god to collapse the wavefunction.

 Jamie Wakeham 12 Feb 2021
In reply to mike123:

It would also be a bit of a shame if Erwin Schrodinger was remembered as Ernest

You're right, though - the Schrodinger equation is a very elegant piece of maths, and it is a pity that his position in popular memory is only due to a poorly understood analogy.

Post edited at 18:57
 Jon Stewart 12 Feb 2021
In reply to cumbria mammoth:

> You say this as if your interpretation is the settled interpretation of quantum mechanics

I can see that it reads that way, my error, but I have no idea whether many worlds or collapse of the wavefunction - or something totally different - is right. There's definitely something funny going on. I just think that if collapse is true, then there are no good reasons to invoke consciousness in how collapse happens.

Post edited at 19:07
 marsbar 12 Feb 2021
In reply to Jon Stewart:

I rather like the many worlds theory.  

 marsbar 12 Feb 2021
In reply to Jon 

No, thanks for that.  Will have a look. 

 alexm198 12 Feb 2021
In reply to Jon Stewart:

Right, good. I think we are mostly on the same page.

The reason I asked whether you thought many worlds was fanciful was that, if you had said 'no, many worlds is far too fanciful too!', I was going to point out that many worlds is a very literal interpretation of QM, and so our intuitions about metaphysical fancy are not much cop for navigating how to think about fundamental physics. I think this is where a lot of people's aversion to slightly spectacular quantum stuff comes from.

Anyway, if I have understood correctly, your objection to conscious collapse is basically that insofar as we need a collapse mechanism at all, the more modest attitude to take is that physics will eventually yield a satisfactory description of collapse. This is contrasted with the more extravagant attitude that we should rush in with all the fairy woo-woo stuff and say that somehow it is linked to the hard problem. 

I very much agree. My original comment was intended simply to point out that in the absence of a conclusive argument that consciousness definitively could not be behind the collapse of the wavefunction, one cannot write it off as bunk, even if it might seem that way. I'm not a believer, but God might come down from on high tomorrow and prove us all wrong, after all. 

> As things stands, you may as well appeal to god to collapse the wavefunction.

I agree. Indeed, why appeal to anyone to collapse the wavefunction? One could simply meet the formalism on its own terms and be a many worlds theorist. 

 Stone Idle 12 Feb 2021
In reply to Shaun mcmurrough:

There’s is in fact a third possible state that Schrodinger, not being a naturalist or cat fancier, completely missed. The cat was was actually absolutely bloody furious.

In reply to alexm198:

> But the wavefunction doesn't collapse in many worlds. My point was that a proponent of many worlds would claim that stipulating some dubious collapse mechanism to save our metaphysical intuitions is more radical than simply taking the quantum formalism at face value (which is what many worlds proponents take themselves to be doing).

I was too eager there to trying to mirror your phrasing in order to make my point so I deserved that to backfire on me.

The wavefunction doesn't collapse in the many worlds interpretation fair enough but to me it seems like the collapse of the wave function is a small matter when compared to the idea that a whole universe is formed every time a particle/wave encounters another particle/wave.

The wavefunction may not even exist in the real world except as a mathematical description in which case its collapse really would be nothing to worry about whereas a many worlds interpretation has some profoundly strange implications such as the real existence of potentially infinite copies of oneself which are splitting off from you at every moment, and the existence of strange universes where the monkeys actually do type out the works of Shakespeare.

Post edited at 22:37
In reply to wercat:

> I believe in radio waves!  They reflect off the ionosphere and they can't just be particles as you would have trouble modulating one over time with, say, a Nigel Farage broadcast.  Accelerate an electron back and forth (point of presence of the electric and magnetic fields, probably a wave too but only chasing its own tail at the speed of light rather than travelling)  and the fields waggle too

> plus the idea of our brains filtering out things that don't fit into our reality is totally unnecessary - our perceptions are based on what our real bodies and component materials can perceive which is levels of abstraction above the quantum level - so it's the only way they can perceive as it is the way they work - no filtering is necessary

Fair enough, I probably need to work on my amateur interpretation but what I am saying is that the universe fundamentally doesn't behave in the way we experience it and I don't think there is much dispute about that.

Radio waves are also particles (photons) so observing that radio waves exist doesn't solve the issue. When we are not watching do they exist as a wave, a photon or this abstract concept of the wavefunction, and when we do observe them has anything really changed or not?

In reply to Jon Stewart:

> I can see that it reads that way, my error, but I have no idea whether many worlds or collapse of the wavefunction - or something totally different - is right. There's definitely something funny going on. I just think that if collapse is true, then there are no good reasons to invoke consciousness in how collapse happens.

Definitely something funny going on. 

> So we've got a few options: we could go for many worlds; or we could say that something about the system where you're making measurements collapses the wavefunction, or you could say that the whole system stays in superposition until a conscious observer notices. 

I hope I'm not taking liberties in quoting this part of your post to alexm198 in isolation but it's a neat tee up to why I think consciousness is necessary.

If it's the many worlds then when does the split occur, is it the same event as causes the wavefunction to collapse? If so then that leaves us in the same place

If it's something about the system we're making measurements in then what's special about the system we've set up? If the wavefunction collapses due to encountering the photons that we are firing at it to measure it, then does the same happen when a photon meets an electron in the void of space? If so then how long does it exist as a particle or wave and when does it return to the form of a wavefunction and why?

I don't think these concepts make sense except in the context of a conscious observer.

In reply to thread..

A lot of reading and all done in a really  interesting way...indeed mind blowing that an atom can exist in two different states..

The hydrogen atom as shown in the clip.

Found this a good visual input..

youtube.com/watch?v=BMIvWz-7GmU&

And then add in our own consciousness and how we perceive the world..that there is this multiple universe with matter in different forms...my head hurts.

 wercat 13 Feb 2021
In reply to cumbria mammoth:

The reason I mentioned radio (remember, Test Match Special has a wavelength of 1500 metres and some signals are kilometers in wavelength is that they actually stretch the idea of a particle beyond what is reasonable.   Yes, your receiving antenna has particle like points of presence of an e/m field in its atoms but they are interacting with a travelling wave immensely longer in wavelength in order to receive the stimulation needed for a current to flow in the antenna.

I find it far more helpful to disregard the idea of a particle and think of a wave which instead of travelling as a single cycle or as part of a chain of cycles of field oscillations is instead travelling at the speed of light in a trapped or "congealed" as Einstein put it around its own uncertain centre, and I think of the Higgs field as that which traps what would travel through space into chasing its own tail at c.

I also like the idea that the entrapment of waves chasing themselves might be evidence of one of the hidden dimensions interacting with what might be loosely described as a "spinning wave".

 john arran 13 Feb 2021
In reply to wercat:

I'm not up with more recent theories but I wouldn't be at all surprised to find that the whole concept of matter turns out to be illusory, in much the same way that Einstein showed gravity to be.

 Jon Stewart 13 Feb 2021
In reply to john arran:

> I'm not up with more recent theories but I wouldn't be at all surprised to find that the whole concept of matter turns out to be illusory, in much the same way that Einstein showed gravity to be.

Quantum field theory pretty much does this. And it gets worse, they're even trying to do away with time...

But the way I see it, if you look at nature in sufficient detail, all you see is patterns that are well-described by maths. Concepts like "matter" or "force" are just mental constructs that relate the real mathematical patterns to our macroscopic experience of the world. Max Tegmark takes this to its philosophical conclusion, that the universe really *is* nothing more than mathematical structure. 

Not read his book, but here's a talk:

https://www.youtube.com/watch?v=_3UxvycpqYo&ab_channel=TheRoyalInstitut...

 wercat 13 Feb 2021
In reply to john arran:

well there is an argument for saying that matter is an experience.

After the years of reading up on what everything is actually made of (beginning actually with trying to understand what radio waves are!) I spoke to the physicist at Berne and asked him if it wasn't possible to think that matter is actually made of "no thing" - in the sense of no thing being what a vacuum is.  He said that this was arguably close to the truth of what matter is - Einsten called it "congealed light".  My epiphany came in a mineral shop in Stanhope where the proprietor showed us a piece of local spar changing colour under a small UV lamp.  He explained about the UV stimulating the emission of visible light from the crystals - I read up on this and realised that effectively the incident UV light became part of the electron before the electron "twanged" and released part of itself as visible light.

Effectively matter is a vacuum behaving badly

 wercat 13 Feb 2021
In reply to Jon Stewart:

> But the way I see it, if you look at nature in sufficient detail, all you see is patterns that are well-described by maths. Concepts like "matter" or "force" are just mental constructs that relate the real mathematical patterns to our macroscopic experience of the world. Max Tegmark takes this to its philosophical conclusion, that the universe really *is* nothing more than mathematical structure. 

 It's quite a trendy way of describing it now but I find it easier to accept a slight tweak to "mathematically describable" rather than just "mathematical" as it is less mystical.

 Jon Stewart 13 Feb 2021
In reply to cumbria mammoth:

> If it's the many worlds then when does the split occur, is it the same event as causes the wavefunction to collapse? If so then that leaves us in the same place

I'd ask Sean Carrol to explain it, I don't understand it well enough to say. My experience in quantum physics amounted to solving the schrondinger equation for a single particle in one dimension as an undergrad; in many worlds you have to consider the wavefunction of the entire universe so I can't quite get my head around how it works.

But apparently, the many worlds interpretation is just a matter of trusting that the wavefunction of the universe evolves over time, and that all the possible realities described by it exist simultaneously. There's nothing special about where it "splits" - that happens at every interaction.

> If it's something about the system we're making measurements in then what's special about the system we've set up? If the wavefunction collapses due to encountering the photons that we are firing at it to measure it, then does the same happen when a photon meets an electron in the void of space? If so then how long does it exist as a particle or wave and when does it return to the form of a wavefunction and why?

This is the point - there aren't good criteria for collapse of the wavefunction.

> I don't think these concepts make sense except in the context of a conscious observer.

But they don't make any more sense with a conscious observer. You can't add things into physics that don't have any physical description. It ceases to be physics when you do that and becomes flannel.

Looking for a way to make QM "make sense" isn't going to turn up much, unless you happen to stumble on the deeper underlying theory that solves its mysteries (which will probably bring about new mysteries, like Russian dolls - this usually happens). Good luck with that

Post edited at 10:26
 mike123 13 Feb 2021
In reply to Jamie Wakeham: thanks . Made the mistake of posting after a couple of fell brewery s fine offerings . 

 mack 13 Feb 2021
In reply to Stone Idle:

> There’s is in fact a third possible state that Schrodinger, not being a naturalist or cat fancier, completely missed. The cat was was actually absolutely bloody furious.

Ah. but cats love boxes, but also hate being boxed in. So there we have a double Shrodinger's cat. The cat must be thought of as both dead and alive but also both content and furious, until the box is opened.

 Lankyman 13 Feb 2021
In reply to mack:

> Ah. but cats love boxes, but also hate being boxed in. So there we have a double Shrodinger's cat. The cat must be thought of as both dead and alive but also both content and furious, until the box is opened.

I had a cat once that would jump into a plastic shopping bag placed on the floor. You could then pick him up in it and even twirl him around. Surprisingly, his wave function didn't collapse and he absolutely loved it.

In reply to Jon Stewart:

> I'd ask Sean Carrol to explain it, I don't understand it well enough to say. My experience in quantum physics amounted to solving the schrondinger equation for a single particle in one dimension as an undergrad; in many worlds you have to consider the wavefunction of the entire universe so I can't quite get my head around how it works.

You're well ahead of me. I'm not one of these that is afraid of maths but it looks beyond me.

> But apparently, the many worlds interpretation is just a matter of trusting that the wavefunction of the universe evolves over time, and that all the possible realities described by it exist simultaneously. There's nothing special about where it "splits" - that happens at every interaction.

>> If it's something about the system we're making measurements in then what's special about the system we've set up? If the wavefunction collapses due to encountering the photons that we are firing at it to measure it, then does the same happen when a photon meets an electron in the void of space? If so then how long does it exist as a particle or wave and when does it return to the form of a wavefunction and why?

> This is the point - there aren't good criteria for collapse of the wavefunction.

Apart from if the criteria is that it happens when the event affects a conscious observer. Then it's pinned down and it seems to agree with what is observed.

> But they don't make any more sense with a conscious observer. You can't add things into physics that don't have any physical description. It ceases to be physics when you do that and becomes flannel.

The physical description of a whole parallel universe springing into existence at every interaction between particles doesn't seem any less dubious than the collapse of the wavefunction as far as I can see. To be fair though, I think I need to do some learning so that I understand that equation better in order to properly evaluate what you're saying here.

From where I am now, and taking what you are saying about the MWI of the Schrodinger equation to be a fair representation of the position, I think we're at a situation where on the one hand we have an important role for consciousness which seems to align more closely with what we observe and intuit, but on the other hand a many worlds interpretation that seems to be a more elegant way to understand the mathematics, but seems to be messy in a lot of other ways, not least philosophically. 

We don't have the physical description for how a conscious observer would collapse the wavefunction but we don't know what consciousness is either. If your position is that consciousness is an epiphenomenon generated within the brain then a role for consciousness in resolving the real nature of the physical universe might look problematic. If you see consciousness as a fundamental property of the universe (as I do) then maybe this can give us a better insight?

> Looking for a way to make QM "make sense" isn't going to turn up much, unless you happen to stumble on the deeper underlying theory that solves its mysteries (which will probably bring about new mysteries, like Russian dolls - this usually happens). Good luck with that

Ha, you're right there. As they say, if QM makes sense to you then you don't understand it properly.

 alexm198 13 Feb 2021
In reply to cumbria mammoth:

> The physical description of a whole parallel universe springing into existence at every interaction between particles doesn't seem any less dubious than the collapse of the wavefunction as far as I can see.

I absolutely understand the intuition behind why you would think this, but I'd point out that there are at least two ways in which the many worlds picture is less dubious than collapse:

First, it doesn't overreach the formalism. This really can't be stressed enough. The starting point for many worlds is simply the observation that the cat superposition really does contain all of the structure of the dead cat state plus all of the structure of the alive cat state. The only reason to introduce a collapse mechanism (and it is introduced, not derived, i.e. it is simply added by fiat) is to reconcile what QM tells us with what our own experience of classical physics. The addition of a collapse mechanism goes beyond the bare quantum formalism. 

Second, it actually suggests a dynamical mechanism for branching (or 'splitting', if you prefer to call it that). It's decoherence. With the exception of the Ghirardi-Rimini-Weber theory, most collapse interpretations offer no physical mechanism for wavefunction collapse. So not only does the collapse mechanism have to be added by hand, in most cases there isn't even a mooted physical process to explain what 'collapse' is supposed to be.

In reply to cumbria mammoth:

I don't have much to add, other than I understand the splitting/branching to occur at *every* interaction of every particle.

Problematic if, as in QM, time is a continuum. But perhaps less mathematically and philosophically troubling if time (or rather spacetime) is itself quantised, as in some the attempts to unite QM with relativity.

 john arran 14 Feb 2021
In reply to Shaun mcmurrough:

I find the many worlds theory just bizarre, and I appreciate that this is in large part due to the fact that it bears no intuitive relation to anything we have ever experienced, or presumably could ever experience, in our perceptive realm.

Also, if new universes are constantly being created, does that mean there are infinitely fewer universes in 'existence' now than say ... now? Could universes ever recombine? My experience of massively exponential growth suggests that something must change very quickly or it cannot be sustainable, so why would universe creation be different?

Finally, could one, or perhaps all, of the alternative spawned universes be what is commonly referred to as 'heaven'?

cb294 14 Feb 2021
In reply to alexm198:

Even if you had to make an arbitrary choice for driving the collapse, the consciousness of an observer seems to me a particularly stupid and implausible pick.

In a way physics still seems to be not much further than when the cat experiment was devised: Despite all its practical success and theoretical progress at the micro scale, QM cannot be fundamental if it does not offer a clear idea of how macro phenomena emerge. There seems to be a huge gap in understanding that cannot be closed by merely "interpreting" the predictions of the existing theory.

CB

 Lankyman 14 Feb 2021
In reply to john arran:

> Finally, could one, or perhaps all, of the alternative spawned universes be what is commonly referred to as 'heaven'?

As an engineering student I always found quantum mechanics a bit 'away with the fairies', probably because I didn't want to wrestle with the understanding. There's one particular phrase in the New Testament I've always found intriguing/inspiring. Jesus says something like 'in my Father's house there are many mansions'. It could be interpreted in lots of ways. It was one of the bits I read to my dad when he was terminally ill. Who knows what the truth is?

 mack 14 Feb 2021
In reply to Lankyman:

You obviously lived in a place where there really was enough room to swing a cat. 

Was he a bit giddy afterwards?

Post edited at 11:15
 Shani 14 Feb 2021
In reply to john arran:

Doesn't the multiverse idea break the Laws of Thermodynamics?

I guess we know the LoT only apply in our universe and breakdown in singularities, so there are exceptions. Nevertheless...

 Jonny 14 Feb 2021
In reply to Jon Stewart:

> Conscious collapse is proposing a physical process

If you're using 'physical process' as shorthand for 'in the world' as opposed to 'in minds', it depends on how collapse is conceived. The quantum Bayesians would couch the collapse as an appraisal of subjective certainty about a state in the world, thereby removing probability from the fundamentals of physics. In this sense, the qBs align with many worlders (see David Deutsch, for example:  youtube.com/watch?v=wfzSE4Hoxbc&), although they differ in the ultimate centrality of probability in their theories.

For the qBs, then, 'consciousness' (insofar as that maps onto to degrees of belief), doesn't effect a change in the world (which I believe is the sort of causation you call 'bunk'), but does govern the collapse of quantum states.

 alexm198 14 Feb 2021
In reply to cb294:

I completely agree with you, and I wouldn't want anything I said above to be interpreted as me advocating a conscious collapse theory, which does seem immensely implausible. My original post was intended merely to highlight that implausible does not mean impossible and that some very smart people have entertained the idea (which of course does not lend weight to it, but does suggest entertaining it as an unlikely option). To the extent that I have a horse in the race at all, I think it's many worlds. 

> There seems to be a huge gap in understanding that cannot be closed by merely "interpreting" the predictions of the existing theory.

I seem to recall from previous discussions that you are generally pessimistic about the value of philosophy of science, and I assume that extends to philosophy of physics, so I can understand why would would think this! But putting that to one side, even if you accept that this gap needs to be closed by physicists, I still think many worlds has the best prospects. Decoherence seems to offer the best way of understanding, dynamically, how classical behaviour is supposed to emerge from the underlying quantum dynamics.

Post edited at 11:33
 Lankyman 14 Feb 2021
In reply to mack:

> You obviously lived in a place where there really was enough room to swing a cat. 

> Was he a bit giddy afterwards?

It was a two up/down terrace so only just! He seemed to get the same enjoyment as you would on one of those theme park rides. Schrödinger wouldn't have had any problems getting him in the box.

 wercat 14 Feb 2021
In reply to Shaun mcmurrough:

For over 20 years or so (I know because the idea came to me in Glencoe in the mid 90s)  I've wondered not about consciousness affecting the state of "particles" but more of the effect on consciousness of infinite-worlds.

We could all be both dead and alive in a superposition of states, all the time.

It seemed to me then that if you experience a moment where you could live or die (Schroedinger's Tat above) then because of the superposition of states you don't collapse a wavefunction so much as experiencing both states.  Because one of the experiences is over quite quickly that which remains of your consciousness would follow the longer timeline and experience the bandwidth universes in which the tat holds and so you would be unaware of your deaths always.  You could therefore experience surviving all the near death moments it was possible to survive until it was not possible for any physical body to survive in any universe.   In other words you always survive near misses in your owm consciousness but other people's consciousnesses take in the universes where any one of these events kills you.  When I told this idea to a fellow climber in Glencoe in over 25 years ago he thought it rather Christian somehow which I hadn't expected.

We also discussed whether the phenomenon of precognition could be based on infinite-universes where timeflow is not the same in all paths and the possibility of our brains being not in one but in a bandwidth of quantiverses and therefore experiencing information from versions with events at different time-points

Post edited at 13:39
 deepsoup 14 Feb 2021
In reply to wercat:

Essentially combining the 'many worlds' theory with the weak anthropic principle - everything that can occur does occur in one of many universes, but you can only experience the universes in which you survive so will always find yourself in one of those.  That was the plot of an Alaistair Reynolds short story in his 'Zima Blue' anthology.  (I forget the title of the story itself.)

The story concerns a physicist who has become convinced of exactly that, but a step further than what you've said because no matter how vanishingly unlikely it is that you'll survive you will, so we're all essentially immortal and not in a good way.

Alastair Reynolds is generally a science fiction writer (and also a proper scientist - a former astrophysicist), but I think it's fair to say the genre of that particular story is more 'horror'.

Post edited at 13:52
 Jon Stewart 14 Feb 2021
In reply to john arran:

> Also, if new universes are constantly being created, does that mean there are infinitely fewer universes in 'existence' now than say ... now?

Yes.

> Could universes ever recombine?

No.

> My experience of massively exponential growth suggests that something must change very quickly or it cannot be sustainable, so why would universe creation be different?

There's nothing unsustainable about the ever-branching wavefunction. It's not using up finite resources, it's just evolving in time with ever more branches like a fractal tree.

> Finally, could one, or perhaps all, of the alternative spawned universes be what is commonly referred to as 'heaven'?

If you like. But it's not a lot of use because the different branches are totally independent, you can't travel or pass information from one to another.

Post edited at 14:13
 wercat 14 Feb 2021
In reply to deepsoup:

sounds like an excellent read - I'll look out for it.

There are at least three stories called "Dark Matter", one an incredibly (and I use the word advisedly and not in the modern usage) atmospheric and creepy story set in an arctic winter on Spitzbergen with a horrifically creepy loss of daylight, another is probably more generic science fiction based on many worlds and a third which is a crime thriller set around Cerne in a science setting by a young female German author - they are all good reads.

 wercat 14 Feb 2021
In reply to deepsoup:

The really creepy thing about the Glencoe convention was that several years later (early 2000s) I saw a cosmology programme discussing the same theory of precognition that I'd come up with in 1995 mentioned by some cosmologist - I nearly jumped out of my skin as I'd never heard of the idea before it synthesised from science I'd been reading in pursuit of understanding Hawking.

Then around 2008 or so in a London bookshop I came across a popular "frontiers of physics" book in Waterstones and there was a chapter based on Russian Roulette that went through precisely the survival scenario I'd talked about in Glencoe - it was really creepy to read.

I suppose parallel synthesis of ideas is common though

 Jon Stewart 14 Feb 2021
In reply to Shani:

> Doesn't the multiverse idea break the Laws of Thermodynamics?

No. The primary objection would be conservation of energy - surely each new universe has just doubled the amount of energy? You just count up the energy so that it's conserved along any one path followed through the branching tree structure, that's how conservation laws work in MW.

There is an apparently good objection to MW which is that it doesn't really solve the measurement problem. Just like Sean Caroll's (more) charismatic defence of MW, it sounds compelling to the partially-educated bystander like me:

https://www.youtube.com/watch?v=kF6USB2I1iU&ab_channel=SabineHossenfeld...

 Jon Stewart 14 Feb 2021
In reply to cumbria mammoth:

> If it's something about the system we're making measurements in then what's special about the system we've set up?

Decoherence is what you're looking for here. I don't really understand it though. To really understand these things, I think you have to actually do physics. When you derive the equations (or follow their derivation as someone else does it on a board in front of you and you try to keep up) and solve them, and your solutions agree with experiment, then you really understand the physics. 

> Apart from if the criteria is that it happens when the event affects a conscious observer. Then it's pinned down and it seems to agree with what is observed.

"It agrees with observation because my theory says that whatever I observe is created the instant I observe it". Kind of hard to falsify a theory that says that!

> The physical description of a whole parallel universe springing into existence at every interaction between particles doesn't seem any less dubious...

It's all highly dubious. FWIW my money is on wavefunction collapse being a "placeholder" for a more fundamental theory - hopefully one which unifies QM and General Relativity. If we're really in business it'll give an account of consciousness too, but I'm not placing any bets on that since I think that's an emergent property of biology not one of fundamental physics (although I do have a bit of a soft spot for the type of panspychism espoused by Galen Strawson and others).

> From where I am now, and taking what you are saying about the MWI of the Schrodinger equation to be a fair representation of the position, I think we're at a situation where on the one hand we have an important role for consciousness which seems to align more closely with what we observe and intuit, but on the other hand a many worlds interpretation that seems to be a more elegant way to understand the mathematics, but seems to be messy in a lot of other ways, not least philosophically. 

Neither is closer to what we observe, they're totally equal in that regard. Intuitions are totally useless here, you can't feel your way through QM going with what "seems right". I think MW is philosophically less problematic than Copenhagen.

> We don't have the physical description for how a conscious observer would collapse the wavefunction but we don't know what consciousness is either. If your position is that consciousness is an epiphenomenon generated within the brain then a role for consciousness in resolving the real nature of the physical universe might look problematic. If you see consciousness as a fundamental property of the universe (as I do) then maybe this can give us a better insight?

You're right about my view of consciousness as epiphenomenal, which is why it looks even more absurd and silly as being instrumental in solving the riddle of Schrodinger's Cat than if I thought it was causally effective at the macro scale. (God help anyone trying to follow this conversation with sentences like that, sorry.). But let's say I'm in the mood for a spot of panpsychism. Great - how does that help? It gives a sort of hand-wavy, flannelly link between QM and consciousness, but that's not physics! 

I go with Searle's Law: when philosophers start talking about quantum mechanics, the density of hot air rises exponentially. (The lovely John Searle, the most sensible philosopher on the planet.)

 Trevers 14 Feb 2021
In reply to deepsoup:

> Essentially combining the 'many worlds' theory with the weak anthropic principle - everything that can occur does occur in one of many universes, but you can only experience the universes in which you survive so will always find yourself in one of those.  That was the plot of an Alaistair Reynolds short story in his 'Zima Blue' anthology.  (I forget the title of the story itself.)

> The story concerns a physicist who has become convinced of exactly that, but a step further than what you've said because no matter how vanishingly unlikely it is that you'll survive you will, so we're all essentially immortal and not in a good way.

> Alastair Reynolds is generally a science fiction writer (and also a proper scientist - a former astrophysicist), but I think it's fair to say the genre of that particular story is more 'horror'.

This follows on from a thought experiment proposed which could in theory allow a brave experimenter to determine between a Many Worlds interpretation of quantum theory or a single-world interpretation such as the Copenhagen Interpretation of Bohr et al. The experiment is called Quantum Suicide, and the experimenter essentially places themself in the role of Schrodinger's Cat.

In one setup, the experimenter points a gun at their head and pulls the trigger. The firing of the gun is conditioned on the measurement of a quantum system - spin up fires a bullet, spin down fires a blank - so with each pull of the trigger, the experimenter has a 50% chance of killing themself. The unfortunate research assistant sees the test subject blow their brains out within a few iterations regardless of which interpretation is correctly, and unfortunately isn't even able to learn anything to  make the PTSD worthwhile.

But assuming Many Worlds is correct, the experimenter can keep pulling the trigger indefinitely and keep firing blanks, with no apparent ill-effects, because they've erased their consciousness in the universe in which the gun fired. With only a small number of pulls of the trigger, they can conclude with high probability (but never complete certainty) that Many Worlds is the correct interpretation.

There are a number of good objections to this possibility. First is that death needs to be instant and simultaneous with the branching of universes, otherwise the observation merely leads to two universes in which the test subject is alive, but one in which they are about to die. Second is that it assumes a direct transference of consciousness, i.e. that current "you" in this universe definitely becomes future "you" in the split universe, not just an exact copy of you, which cannot be proven in advance of the experiment.

One might take the logic of the experiment further and conclude that it is impossible to die full stop, since cell death will be ultimately the result of processes in quantum systems, thus leading to the horrifying possibility of immortality in a dying universe. However this is an even further logical stretch because it assumes that death is a binary process, whereas even cell death is based on upon a macroscopic ensemble of quantum systems, and the death of consciousness must be assumed to be the result of a statistical process of an ensemble of cells. The idea that the quantum state of a single atom or electron is the difference between full living consciousness and death seems absurd. So I think that the immortality horror future is not worth losing sleep over.

 Jon Stewart 14 Feb 2021
In reply to Jonny:

> If you're using 'physical process' as shorthand for 'in the world' as opposed to 'in minds', it depends on how collapse is conceived.

I don't quite get what a "physical process in minds" would mean. I think that minds are what brains feel like from the inside, so anything happening in a mind is also happening in the world (although the things happening aren't identical, just perfectly correlated).

> The quantum Bayesians would couch the collapse as an appraisal of subjective certainty about a state in the world, thereby removing probability from the fundamentals of physics. In this sense, the qBs align with many worlders (see David Deutsch, for example:  youtube.com/watch?v=wfzSE4Hoxbc&), although they differ in the ultimate centrality of probability in their theories.

Thanks, I'll have a go with that, I like Deutsch. Don't know anything about what quantum Bayesians say.

 Pbob 14 Feb 2021
In reply to Shaun mcmurrough:

I can explain it. Or maybe I can't explain it. Nobody will know until I open my mouth and try.

 Jonny 14 Feb 2021
In reply to Jon Stewart:

> I don't quite get what a "physical process in minds" would mean. I think that minds are what brains feel like from the inside, so anything happening in a mind is also happening in the world (although the things happening aren't identical, just perfectly correlated).

Absolutely, I too share that philosophy of mind, although some are certainly happy speaking of physical mind-brains. Either way, my point was that a consciously caused collapse isn't necessarily bunk if the collapse is conceived as a change in uncertainty (that is, a state localised in the perceiver and not in [the rest of] the world), as per the qBs.

 Jon Stewart 14 Feb 2021
In reply to Jonny:

I couldn't quite follow deutsch's argument in that lecture. It all started off nice and paletable, then he lost me. 

I'll try to find a few more layman's qB bits on YouTube to see if I can get the idea. I should be in for a headstart, I've studied a bit of both QM and Bayes' theory. 

 deepsoup 14 Feb 2021
In reply to Trevers:

> thus leading to the horrifying possibility of immortality in a dying universe

Well, yes, that's the underlying horror that makes the short story I'm talking about a horror story.

I guess the 'quantum suicide' experiment you're describing is a variation on Schroedinger's Cat as a thought experiment, this is a little different though - more a sequel than an alternative version. 

The cat shows us the paradox of trying to scale up this 'quantum' business to the macro world of our everyday experience, having done that we're back in the macro world and there's no need for the fancy equipment to conduct the 'quantum suicide' experiment.

In 'many worlds', at the moment that the cat lives/dies the world splits in two.  In one the cat is alive, in the other it's dead.  Five minutes later there are a near infinite number of worlds, in just under half of which the cat is alive.  Other cats are alive or dead in various combinations in all those other worlds, and everything that could have happened in the preceding five minutes has, in at least one of those worlds.

In the experiment you describe, with the 50/50 chances of getting a blank versus a bullet you say you'd need to pull the trigger a lot, each time becoming a bit more convinced.  But if that's the aim, why waste time with 50/50 odds.  You could flip a coin four times in a row, or you could just get the same result by rolling a 16-sided die and seeing if a 1 comes up.  Or you could toss a billion coins simultaneously.  Lose the fancy gear, load a live bullet in every chamber.  If you're the version of yourself living in the world in which you survive the gun won't fire, or you'll sneeze and miss, maybe the cat that is still alive will jump up and knock your arm away at just that moment.  Anyone one of a vast number of scenarios in which you survive will play out, which in turn are a small subset of the scenarios that could play out, in most of which you shoot yourself and die.

> the death of consciousness must be assumed to be the result of a statistical process of an ensemble of cells

Sure.  But we're discussing statistics now.  We're not splitting one world into two with the toss of a coin, we're splitting in into a vast but finite number of worlds with the simultaneous toss of many billions of coins.

> Second is that it assumes a direct transference of consciousness, i.e. that current "you" in this universe definitely becomes future "you" in the split universe..

Sure.  Or perhaps more to the point looking back - I made a stupid decision a while ago and was nearly killed as a result, and now I'm just assuming that I'm the same 'me' who I remember getting away with it.  (And not a Boltzmann Brain or something that only thinks it's me, but ugh, lets not get into that.) 

The anthropic principle isn't about where I'm going, it's about where I came from - I'm just one of an unimaginably large but finite number of 'me's living in the worlds in which I made it this far, an infinitesimal subset of the unimaginably larger number of worlds in which I didn't, or never existed in the first place.

> So I think that the immortality horror future is not worth losing sleep over.

Sure.  It's just an interesting thought experiment that I'd never heard of until I read it in the form of a slightly disturbing short story. 

The thing about that quantum suicide experiment really though is that there's no need to conduct it at all.  Not only do you not need the fancy gun that fires a real bullet or not based on the state of a sub-atomic particle, you don't need any gun at all.  We're all committed to the experiment already, just wait.

In reply to marsbar:

Absolutely. He was pointing out how ridiculous current theories (of the time) were by applying them to cats.

 tlouth7 15 Feb 2021
In reply to Weekend Punter:

Quantum entanglement comes about because of a need to conserve various physical properties (called quantum states). So if two particles are created out of an interaction they must have opposite values of those properties such that the total value equals 0 (there are more complex cases e.g. with more than two particles).

Imagine a pair of ice skaters on a rink. They start together and push away from each other so they go sliding away from each other. They started with 0 momentum (because they were both stationary), so after they start moving apart their total momentum must still be 0 (momentum is conserved). If they have the same mass (lets say they do) then they must be moving at equal speeds in opposite directions. The momentum of the two skaters is entangled.

Now imagine you measure the speed of one of the skaters. Because you know their speed, you instantly know the speed of the other skater (remember equal and opposite). It seems like some information about the other skater has travelled to you instantly, but it hasn't really because you already knew everything you needed to work it out.

Further, the entanglement doesn't let you communicate. People think that you could change the properties of one of the particles, and the other one would change instantly to match. But think about our skaters, if you pushed the skater nearer you the other skater wouldn't suddenly lurch. Instead you would break the entanglement such that knowing about one skater's momentum no longer tells you about the other's. So there is no way of sending a signal from one skater/particle to the other through their entanglement.

I hope that makes sense.

 Rob Parsons 15 Feb 2021
In reply to tlouth7:

> Quantum entanglement comes about because of a need to conserve various physical properties (called quantum states). So if two particles are created out of an interaction they must have opposite values of those properties such that the total value equals 0 (there are more complex cases e.g. with more than two particles) ...

I think that is a misleading analogy.

In the quantum case, we do not 'know' the state of either particle - and we specifically do not know the state of local particle - until we measure it. Indeed, until that measurement is taken, both the local and the remote particles have no actual state: they are literally wave functions, not particles, which are in a superposition of all possible states.

At the time the measurement is taken however, that act of measurement finally determines the state of of the local particle - and simultaneously pins down the state of the remote one as well.

That's the spooky 'action at a distance' aspect.

 tlouth7 15 Feb 2021
In reply to Rob Parsons:

I agree fully with your points. I was thinking mostly about trying to explain why entanglement cannot be used to communicate faster than light. I guess I did imply hidden variables which we know are not real.

In reply to .....

From Schrodinger's cat ...to this..

youtube.com/watch?v=TVorG6_csSA&

And then photosynthesis...100% energy transfer.....

I can't get enough..amazing subject.

 Trevers 16 Feb 2021
In reply to deepsoup:

> Well, yes, that's the underlying horror that makes the short story I'm talking about a horror story.

I wasn't intending to put the story down in any way, I just thought it would be an interesting extra discussion for the thread.

> I guess the 'quantum suicide' experiment you're describing is a variation on Schroedinger's Cat as a thought experiment, this is a little different though - more a sequel than an alternative version.

Indeed, they have different goals. As you say, the cat is intended to demonstrate what Schroedinger saw as the absurdity of QM but the experiment itself is inherently pointless because it doesn't differentiate between competing interpretations. The Suicide experiment is intended (hypothetically at least) as a possible experiment to choose the correct interpretation.

> In the experiment you describe, with the 50/50 chances of getting a blank versus a bullet you say you'd need to pull the trigger a lot, each time becoming a bit more convinced. 

10 shots would already give you a less than 1/1000 chance of surviving.

> But if that's the aim, why waste time with 50/50 odds.  You could flip a coin four times in a row, or you could just get the same result by rolling a 16-sided die and seeing if a 1 comes up.  Or you could toss a billion coins simultaneously.  Lose the fancy gear, load a live bullet in every chamber.  If you're the version of yourself living in the world in which you survive the gun won't fire, or you'll sneeze and miss, maybe the cat that is still alive will jump up and knock your arm away at just that moment.  Anyone one of a vast number of scenarios in which you survive will play out, which in turn are a small subset of the scenarios that could play out, in most of which you shoot yourself and die.

The original experiment requires that death is instantaneous with the observation of the quantum system, otherwise the macroscopic death is decoupled from the quantum measurement. If there's any delay, you simply end up with two copies of yourself in two universes, one of whom survives and one of whom is just about to die. I think it's a bit of a stretch to suggest that at the instant you die, your consciousness jumps into a universe in which you're still alive, and I'm not prepared to test that hypothesis!

> The anthropic principle isn't about where I'm going, it's about where I came from - I'm just one of an unimaginably large but finite number of 'me's living in the worlds in which I made it this far, an infinitesimal subset of the unimaginably larger number of worlds in which I didn't, or never existed in the first place.

But the experiment only makes sense in the forwards direction. Every copy of you in every parallel universe will have an seamless memory of their life history, but it doesn't necessarily follow that current you will experience a seamless transition into your future parallel selves. The suicide experiment only works (if it works) because the test subject doesn't enter alive into the universe in which they died. If you put a gun to your head and pull the trigger this instant, your consciousness doesn't jump at the instant of death into another universe in which you decided that was silly, or where you managed to administer a non-fatal shot.

> The thing about that quantum suicide experiment really though is that there's no need to conduct it at all.  Not only do you not need the fancy gun that fires a real bullet or not based on the state of a sub-atomic particle, you don't need any gun at all.  We're all committed to the experiment already, just wait.

I've already scared myself shitless over the thought once, you can't get me a second time

 wercat 16 Feb 2021
In reply to Trevers:

> 10 shots would already give you a less than 1/1000 chance of surviving.

 Not really - I met a soldier once who'd been shot through the head in NI - he did have some mental problems but he was still alive and talking to me

this of course doesn't detract from your thrust

>I think it's a bit of a stretch to suggest that at the instant you die, your consciousness jumps into a universe in which you're still alive, and I'm not prepared to test that hypothesis!

When I considered the idea 25 years ago it seemed to me to be possible that consciousness might spread across a "bandwidth" of very closely "in phase and near-in phase universes" so far from jumping into others information flow from the ones in which you don't survive would cease

I already had this in mind when I thought up this and different time flows to explain the phenomenon of precognition that I've  suffered startlingly from all my life and which is really spooky - I was trying to come up with science that might explain what I have experienced.  Cases where the usual dismissals simply could not apply

Post edited at 09:14
 deepsoup 16 Feb 2021
In reply to Trevers:

> I think it's a bit of a stretch to suggest that at the instant you die, your consciousness jumps into a universe in which you're still alive..

But nobody who proposes 'many worlds' seems to struggle with that when the cat dies (and lives) in the original experiment.  You're conscious in the one world in which you put the cat into the box, and also in the two separate worlds in which you take the dead/alive cat out of the box again. 

After the experiment, both versions of 'you' have identical memories up to the point that you opened the box and observed the state of the cat.  The cats are identical too, except that one is alive and the other is dead.

Edit:
I originally said "the cat dies (or lives)", but of course it's both!

Post edited at 10:42
 deepsoup 16 Feb 2021
In reply to wercat:

OT but interesting - have you heard of Phineas Gage?  He was an American railroad worker who had a massive iron rod driven right through his head in a blasting accident in 1848 and survived for twelve years afterwards.
https://en.wikipedia.org/wiki/Phineas_Gage

> When I considered the idea 25 years ago it seemed to me to be possible that consciousness might spread across a "bandwidth" of very closely "in phase and near-in phase universes" so far from jumping into others information flow from the ones in which you don't survive would cease

Woah!

There is a much simpler possibility to consider - that our own individual consciousness isn't even a thing.  It's just an illusion man.  Ommmmmm!

In reply to deepsoup:

We each experience our own individual consciousness (or at least I know I do) so that is the only thing that anyone can be absolutely certain of.

The simplest possibility is to consider that our individual consciousness is the only real thing and that space, matter, and time are illusions.

 Shani 16 Feb 2021
In reply to cumbria mammoth:

> The simplest possibility is to consider that our individual consciousness is the only real thing and that space, matter, and time are illusions.

Or a simulation? Is it time to introduce the concept of Roko's Basilisk?

 wercat 16 Feb 2021
In reply to cumbria mammoth:

but our consciousness is not the clever part of us, probably not even the majority part of us, whatever fibs yours tells you - it's like a kid with a toy driving wheel in the back seat thinking its controlling the car

It is no more in control than Skimbleshanks is of the Night train

Post edited at 18:59
 Trevers 17 Feb 2021
In reply to deepsoup:

> But nobody who proposes 'many worlds' seems to struggle with that when the cat dies (and lives) in the original experiment.  You're conscious in the one world in which you put the cat into the box, and also in the two separate worlds in which you take the dead/alive cat out of the box again. 

I'm not entirely sure what point you're making. The death of one person is not problematic for an observer in any interpretation of QM (unless they have empathy, and I don't know if that's a quantum property). The cat experiment plays out the same regardless and the result comes as no surprise.

> After the experiment, both versions of 'you' have identical memories up to the point that you opened the box and observed the state of the cat.  The cats are identical too, except that one is alive and the other is dead.

True, neither version is aware of the other (unless wercat's hypothesis is correct) and both experience a seamless history. But the suicide experiment requires that current you experiences at least one future fork, which is not the same thing. If I'm not explaining myself that well, a similar conundrum can be constructed around the concept of gaining immortality by uploading the contents of your brain into an AI robot at the point of your death. Is there really a continuation of your consciousness into your new host, or have you simply created a copy that thinks it's you and remembers being you? Hopefully that makes sense.

> I originally said "the cat dies (or lives)", but of course it's both!

Not according to Many Worlds, the superposition never exists

Post edited at 00:31
 deepsoup 17 Feb 2021
In reply to Trevers:

> I'm not entirely sure what point you're making.

> True, neither version is aware of the other (unless wercat's hypothesis is correct) and both experience a seamless history. But the suicide experiment requires that current you experiences at least one future fork, which is not the same thing.

Ah, that's the point I'm trying to make and where we're at cross purposes! 
I'm not with you at all on how they are not the same thing.

Forgetting the suicide thing for the moment, I'm talking about the human experimenter in Schroedinger's original experiment with the cat.

> Not according to Many Worlds, the superposition never exists

Cool.  But even if it does exist while the box remains closed, I'm talking about a time just after the box has been opened and the state of the cat observed by the human.

Does the person who puts the cat into the box not go on to experience at least one 'future fork' in which the experiment is over and the cat has been taken out again?

An hour later, the physicist needs to feed the cat (or bury the cat) - is that not the same person who put the cat into the box earlier that day? 

If there never was a superposition and the cat is still both alive and dead, the physicist has to both feed the cat and bury it..  Two cats (one alive, one dead), two physicists (both alive), two worlds!  Which physicist is the same person who came into the lab and set the experiment up that morning - either, neither or both?

If it's neither, shouldn't the physicist approach the experiment with the cat with the same trepidation as the 'suicide' experiment?

>  a similar conundrum can be constructed around the concept of gaining immortality by uploading the contents of your brain into an AI robot at the point of your death.

I almost mentioned the same science fiction conundrum in Star Trek terms.  (Every version of Star Trek has had a play with the concept as a plot device at some point.) 

The 'transporter' measures everything about a person completely (working around the uncertainty principle because it's fitted with a 'Heisenberg Compensator'), then it simultaneously annihilates them completely, transmits the information and reassembles them in a different place.

So there you are in orbit on the Enterprise, beaming down to San Francisco for a meeting at Starfleet HQ.  Standing on the pad you hear the sound effect, see the shimmer, but when it stops you're still standing there.  The operator says "Don't worry Sir, it's just a temporary glitch at this end.  You have already arrived in San Francisco and are on the way to your meeting.  If you could just wait patiently on the pad while the system resets itself, you'll be annihilated in just a moment."

 Jon Stewart 17 Feb 2021
In reply to cumbria mammoth:

> The simplest possibility is to consider that our individual consciousness is the only real thing and that space, matter, and time are illusions.

That makes things *a lot* more complicated - you've now got to explain why there *appears to be* a whole external reality we all share. How is this data being generated? If solipsism is right, who has created the David deutsch lecture I can't understand? Another part of my consciousness that's cleverer than the part of my consciousness I'm conscious of? Hang on, we've already added something outside my consciousness...

In reply to Jon Stewart:

> That makes things *a lot* more complicated - you've now got to explain why there *appears to be* a whole external reality we all share. How is this data being generated? If solipsism is right, who has created the David deutsch lecture I can't understand? Another part of my consciousness that's cleverer than the part of my consciousness I'm conscious of? Hang on, we've already added something outside my consciousness...

I was just throwing that one out there. I do believe in an external reality even if it might not exactly match up with our every day experience of it. A universal consciousness, of which our individual consciousness is just part of a greater whole, might solve your objection to that idea though?

I've been meaning to come back to you actually as I've been looking at the MWI and decoherence. I'll try to bring it together when I get a chance tonight. 

 Dominic Green 17 Feb 2021
In reply to Phil Lyon:

> Voldemort?

> I was thinking about Schrodinger's Alarm Clock this morning. You have no idea what the time is, the night could be endless, but as soon as you look at the alarm clock, it's about to go off.

Schrodinger's lockdown too

 gld73 17 Feb 2021

Moving off QM topics from Schroedinger's Cat to Heisenberg's Uncertainty Principle for a moment...

A police officer pulls over a speeding motorist. "What's your name?"

"Werner Heisenberg"

"Well Mr Heisenberg, can you tell me what speed you were doing?"

"No officer, I can't... but I can tell you exactly where I am".

In reply to alexm198: Jon Stewart:

I've been looking at this Many Worlds Interpretation and Decoherence. I'm very aware that following the logic of the equation, even where it goes against our every day experience, has opened up a deeper understanding of the truth before.

The Schrodinger equation describes a wavefunction where all possibilities for a quantum system actually exist in this universe. It doesn't provide any mechanism for any sort of split in the universe. Decoherence as far as I can make out is not a mechanism for how worlds split but is an explanation for why we can't observe the other possibilities and it is not a sudden fork but a more gradual (on the quantum scale) untangling of threads of reality.

I think for this type of interpretation to have merit you have to abandon your view of a definite distinct reality. Instead you'd have to view a particle/ a quantum system/ an object/ the universe as existing as a sort of smear of all the possibilities at once, not in many universes but all in this one.

It's not that there are other versions of you that made different decisions in parallel worlds, those shadow you's are real in this world but can't be observed or have any effect on the strand of reality that you are experiencing.

It fits the equation but it has some shocking implications for what reality and the nature of self means. These concerns are aesthetic not scientific but so is the objection to conscious observation as the cause of the wavefunction collapse. The wavefunction collapse and our definite reality is what is actually observed and, as both interpretations predict the same results it can only be only a matter of taste to prefer one over the other.

 JimmAwelon 17 Feb 2021
In reply to Shaun mcmurrough:

Chwarel Bach y PenrhynSchrödinger's Cat (3a)

So now I know about the theory what does it say about one of the easiest lines at my most local crag? 

In reply to wercat:

Your quantum suicide idea places consciousness in prime position in the many worlds interpretation as well. If consciousness is able to leap between the various strands of reality then it really would be privileged over every other physical phenomenon in the universe.

The idea you put forward would be a lonely existence though. If you are only experiencing consciousness in one of the strands of reality which contains a version of you at any one time, then you think of the sheer number of splits that there must be in the MWI (your reality splits not just each time you make a decision or observation but also each time a particle interacts with another particle on a star at the other side of the galaxy), then the chances of existing on a thread of reality that contains another conscious mind must be infinitely small.

My philosophical objections won't make any difference to what the truth really is but I find it hard to accept that one.

A many worlds type of interpretation I could probably get along with is where all the possible realities do exist in some sense but consciousness (for everyone) follows only one of the threads of reality.

 wercat 18 Feb 2021
In reply to cumbria mammoth:

I consider that our minds (of which consciousness is only a foamy dream on the top) might well be riding a bandwidth of closely related worlds - as I described there is no jump between worlds if you do something that kills you in part of that bandwidth - input to your brain from those worlds would cease while it continued from those in which you are still alive.  This is an application of an idea of bandwidth of worlds (call it co-channel reception in radio terms) which I evolved after a great deal of reading to explain lifelong startling incidences in my own life of information appearing apparently from no-where - the idea of related worlds having slightly different time-flow allows for this if the brain experiences contact with more than one world-channel. The usual tired argument of my consciousness being unaware of what my subconscious has noticed and knows simply can't apply to some of these instances of cognition.

There is no such unique privilege of which you speak in what I am saying - it is the effect of many worlds on how our brain works.  BTW you have to disengage conscious thought and have a pretty vacant mind for this stuff to appear - conscious thought seems to block it

btw - it wasn't me who mentioned suicide - my concept - as we were having this discussion in a winter mountain setting was actually was conservation of life in sutuations where you might die or live.

Post edited at 09:31

New Topic
This topic has been archived, and won't accept reply postings.
Loading Notifications...